LSAT and Law School Admissions Forum

Get expert LSAT preparation and law school admissions advice from PowerScore Test Preparation.

 Administrator
PowerScore Staff
  • PowerScore Staff
  • Posts: 8948
  • Joined: Feb 02, 2011
|
#22802
Complete Question Explanation

Weaken. The correct answer choice is (E)

The argument here is that high salaries for politicians is a bad idea, because it draws people who primarily care about money. Since the stimulus is followed by a weaken question, we should seek the answer choice which either shows that such bad people are not actually drawn to politics, or perhaps that the salaries of politicians have a different effect than perceived by the author of the passage.

Answer choice (A): The fact that some politicians help to finance their own campaigns would not make it any less or more likely that they entered politics based on financial motivation, so this answer choice is incorrect.

Answer choice (B): The length of the term of office has absolutely no effect on the strength of the argumentation in the stimulus, so this answer choice is also incorrect.

Answer choice (C): The number of people drawn to politics by the high salaries is not relevant to the central question of whether or not high salaries tend to draw those who are motivated primarily by financial considerations.

Answer choice (D): This answer choice deals with exactly the kind of politicians that this author appears to be concerned about. The fact that those who rely on politics are the ones who are selfish does not provide any insight into the central question of high political salaries drawing the wrong sorts of people.

Answer choice (E): This is the correct answer choice. If this is true, and the politicians tend to have more lucrative options in other fields, there the lure of politics must be based on something other than simple financial motivation.
 yrivers
  • Posts: 68
  • Joined: Mar 15, 2017
|
#33954
Hi,

Can you clarify why C is incorrect? If one is no more likely to enter politics whether it's high or low salary, doesn't that state that one isn't entering politics for the money? If you get a low salary and you still choose to do it?

Thanks,
Yaesul
 Robert Carroll
PowerScore Staff
  • PowerScore Staff
  • Posts: 1819
  • Joined: Dec 06, 2013
|
#33963
Yaesul,

Answer choice (C) is a tricky one! Note that your explanation of it is slightly off. It says that no more people compete, but it doesn't say anything about the identity of the people that compete in the two situations. Thus, it does not imply that "one is no more likely to enter politics whether it's high or low salary." Let me explain why.

Imagine we pay politicians a lot. 10 people run for office. 7 of those people are doing it just because they want the high salary.

Imagine we don't pay politicians a lot. Initially, only 3 people run for office, because the 7 people that would have run in the other situation don't bother - they don't think they'll make much money if they win. However, with these money-hungry people not running, 7 other people decide that it's now worth trying, because the field's not as crowded as it would have been. So we end up with 10 people running for office again. It's not the same 10 people, though! If we paid politicians a lot, it would be more likely to include people in it for the money. When we pay politicians a small amount, we don't get those same people running, but the numbers are still the same. Thus, it's possible that "no more people compete" when officeholders are paid well vs. poorly, yet the argument's point still stands - high pay seems to attract some greedy candidates!

This is why what you said is slightly off. Some people ARE more or less likely to run based on the pay, even if the total number competing doesn't change, because my situation is consistent with that fact. This is why answer choice (C) does not weaken.

Robert Carroll
 martinbeslu
  • Posts: 49
  • Joined: Aug 09, 2017
|
#42446
I don't understand the explanation for why answer choice C is wrong. This seems like a perfect weaken answer.

The question asks what would weaken the argument that:
C E
paying high salaries ---> disastrous effect on the level of integrity among politicians

scenario:
politician salary $50,000... 10 candidates apply
politician salary $55,000... 10 candidates apply
politician salary $60,000... 10 candidates apply
politician salary $100,000... 10 candidates apply
politician salary $1,000,000... 10 candidates apply

The pay is clearly NOT what attracts people to this job. This weakens the argument that a higher salary draws more people into the election. If there is not even a slight increase in the size of the candidate pool when the salary is doubled how can you make the argument that there is a "disastrous effect". I understand that some non-money hungry candidates could drop out of the race if it got much more competitive but it would be a huge stretch to say that every new money hungry candidate displaced exactly one non-money hungry candidate and the total interest in the position increased by zero.

In order to say that this statement does not weaken the authors conclusion you would have to assume that a higher salary dissuades non-money hungry politicians at the same rate or greater than the rate that the higher salary persuades money-hungry politicians. It's just not logical for the total size of the competition pool to stay exactly the same UNLESS money is not actually the motivator for politicians in Valitania. This weakens the argument.

To me, this comes about as close as you can get to proving that high salaries are not the motivation for people to run for office. The candidate pool didn't grow by even one person? And as soon as the salary goes from $50,000 to $100,000 none of the original candidates are interested any more?
 Jennifer Janowsky
PowerScore Staff
  • PowerScore Staff
  • Posts: 90
  • Joined: Aug 20, 2017
|
#42466
martinbeslu,

I definitely had to think long and hard about this one! Here are my thoughts:

Although answer (C) does show that money does not change the number of people running, it does not necessarily say anything about the type people who are running--although there may be the same number, what are their motivations? Answer choice (E), however, does tell us about Valitanian officials--it tells us that those in office in Valitania could have made more money elsewhere, and therefore money couldn't be their biggest motivation. In this case, the argument is about defining their motivations, and (E) does this more clearly.

I hope this helps!
 NolaB
  • Posts: 6
  • Joined: Jan 29, 2018
|
#43885
My question is more about strategy. This one really slowed me down, because I had a hard time zeroing in on what conclusion I was meant to be attacking. I understand that answer choice (E) weakens the argument that high salaries attract people motivated primarily by making money. However,I had trouble identifying the conclusion before proceeding to the answer choices.

At first I thought that I was supposed to weaken the assertion that high salaries have had a disastrous effect on the level of integrity among politicians. Then I thought I should attack the idea that "the wrong people" (motivated primarily by money) must have been attracted into politics. In the end, you end up attacking an argument that seems to combine both elements? I run into this issue a lot when identifying the conclusion, and maybe I'm overthinking things. How do you know, in a case like this, where the conclusion stops and starts?

I hope that makes sense. I'm sorry to get so nitty-gritty, but I'm really trying to dig into why this stuff is keeping me from moving quickly and accurately through the questions.

Best,

Nola
 Francis O'Rourke
PowerScore Staff
  • PowerScore Staff
  • Posts: 471
  • Joined: Mar 10, 2017
|
#43894
Hi Nola,

You may still be incorrectly identifying a premise as the conclusion. As the administrator wrote above,
The argument here is that high salaries for politicians is a bad idea, because it draws people who primarily care about money.
The idea that "high salaries attract people motivated primarily by making money." is a premise of the argument. The speaker introduced this idea with a clear premise indicator "This is because...."

This particular stimulus was relatively difficult to analyze because there is an intermediate conclusion present in the argument:
..... Premise: prospect of earning high salary attracts greedy people.
..... Intermediate Conclusion: greedy and selfish people have entered Valitanian politics.
..... Main Conclusion: paying high salaries has had a bad effect on the integrity of Valitania.

After we identify a premise, we should treat it as a fact and not seek to weaken it. Rather, we should zero in on weakening what the speaker thinks follows from that premise.

As for strategy, it seems like you are getting the real difficulty. The answer choice is not going to outright contradict that conclusion by saying "actually the high salaries have been great for Valitania! The high salaries have attracted wonderful people into politics!" That would be too obvious and easy for a number 19 question.


Answer choice (E) affirms the premise 'earning a high salary attracts greedy people', but it weakens the connection between that premise and the conclusion. That is, if we believed answer choice (E), would the conclusion that Valitanian politicians have gotten worse still follow? Well, if most of these politicians could earn more elsewhere, do we really believe that they must be greedier and more selfish?
 NolaB
  • Posts: 6
  • Joined: Jan 29, 2018
|
#43905
Aha! You know what it is? It's silly, but I think I've been trying to pick out the conclusion as an entire sentence in the stimulus, rather than noticing when we switch between premise and conclusion mid-sentence. Let's say a premise follows a conclusion in a format like this:

Valentines day is good for relationships because most couples give each other chocolate.

The conclusion and the premise are in the same sentence, but are obviously two separate components of the (pretty weak) argument, right? You weaken the argument by weakening the link between the premise (couples give each other chocolate) and the conclusion (Valentines day is good for relationships), but you don't "attack" the whole sentence, because it's not the conclusion! You take the premise to be true and consider it separately. Am I on the right wavelength?

This seems obvious when you're looking at a simple sentence or even a few sentences, but I guess sometimes I get bogged down in the details of the stimulus and forget to check for basic indicators. Thank you!
 Emily Haney-Caron
PowerScore Staff
  • PowerScore Staff
  • Posts: 577
  • Joined: Jan 12, 2012
|
#43915
Hi NolaB,

I think you hit exactly on what was tripping you up. Your example is a perfect illustration, and your reasoning is spot on! To weaken your sample argument, we might have an answer choice that read, "Couples argue more after they've received gifts from each other." Or, "Giving trivial gifts like chocolate has no impact on relationship quality."

I am so impressed with your reasoning and your determination to identify right where the problem was for you. Great work!!
 Tomars
  • Posts: 15
  • Joined: Aug 03, 2017
|
#49487
I debated between C and E. I understand why C is wrong now. Even though, the same number of folks compete when the salary is high or low, we do not necessarily have the same types of people. As the text says, it's still quite possible to have "the wrong [type] of people" who are more interested in making money.

But I'm still having problem justifying E because of the word "most". Even if "most" (e.g., 52%) current politicians could have been better paid elsewhere, there could still have been a significant enough number of folks (e.g., 48%) who joined because of the high pay and who could not as easily obtain better paying jobs elsewhere. In that case, it's still quite possible that the "wrong" type of people are drawn to being politicians in Valitania..those attracted primarily to the high salary.

And I imagine the burden of proof for this question type is not lower/has to be air tight, since it does not ask for which answer "most weakens".

Get the most out of your LSAT Prep Plus subscription.

Analyze and track your performance with our Testing and Analytics Package.